mike and jed went skiing at 10:30 a.m. they skied for 1 hour and 40 minutes before stopping for lunch. at what time did mike and jed stop for lunch? answer

Answers

Answer 1

Mike and Jed stopped for lunch at 12:00 p.m. after skiing for 1 hour and 40 minutes.

To add time, we need to convert minutes to hours and minutes. There are 60 minutes in an hour, so we can divide the number of minutes by 60 to get the number of hours and the remaining minutes. In this case, 1 hour and 40 minutes is the same as

(1 hour + 40/60 hours) = 1.67 hours

So, to find out what time they stopped for lunch, we can add 1.67 hours to the starting time of 10:30 a.m. We can do this by converting 10:30 a.m. to 24-hour format, which is 10:30. We then add 1.67 hours to 10:30, which gives us a total of 12:00 p.m. (or noon).

To know more about time here

https://brainly.com/question/4199102

#SPJ4


Related Questions

heights for a certain age are normally distributed, with a mean of 40 inches and a standard deviation of 5 inches. use the empirical rule to find the lower and upper boundaries for the middle 68% of heights.

Answers

The lower and upper boundaries for the middle 68% of heights falls between 35 inches and 45 inches.

The empirical rule, too known as the 68-95-99.7 rule, could be a statistical rule that depicts the approximate rate of perceptions that fall within a certain number of standard deviations from the mean of a normal distribution. Particularly, the empirical rule states that for a normal distribution, around:

68% of the observations fall under one standard deviation of the mean.95% of the observations fall under two standard deviations of the mean.99.7% of the observations fall under three standard deviations of the mean.

According to the Empirical rule,

Mean = 40 inches(given)

Standard deviation = 5 inches(given)

To find out the lower and upper boundaries for the middle of 68% heights-

lower boundary = mean - standard deviation

upper boundary = mean + standard deviation

So,

lower boundary = 40 - 5 = 35 inches.

upper boundary = 40 + 5 = 45 inches.

Therefore, 68% of heights fall between 35 inches and 45 inches.

To learn more about empirical rule,

https://brainly.com/question/30573266

The three characteristics required to properly describe a sampling distribution are _____
✓ 1. mean 2. variance 3. shape

Answers

The three characteristics required to properly describe a sampling distribution are mean, variance, and shape.

1. Mean: The mean, also known as the average, represents the central tendency of the sampling distribution. It is calculated by adding up all the sample means and dividing the sum by the total number of samples.

2. Variance: The variance is a measure of how much the sample means vary from the overall mean of the sampling distribution. It helps determine the spread or dispersion of the data.

3. Shape: The shape of a sampling distribution describes its general appearance. Common shapes include symmetrical (e.g., normal distribution) and skewed (e.g., positively or negatively skewed) distributions. The shape can give insights into the underlying population and the behavior of the data.

By considering the mean, variance, and shape, you can effectively describe the characteristics of a sampling distribution.

Learn more about  quantity variance here brainly.com/question/14116780

#SPJ11

Select the correct answer. Which value of x from the set {4, 5, 6, 7}, makes this equation true? 4(8 − x) = 8 A. 4 B. 5 C. 6 D. 7

Answers

Answer: c. 6

Step-by-step explanation:  

Option-C is correct that is 6 number from the set {4 , 5, 6, 7} makes the equation 4(8 - x) = 8 true.

Given that,

We have to find which value of the x from the set {4 , 5, 6, 7} makes the equation 4(8 - x) = 8 true.

We know that,

What is a set?

A group of well defined objects is referred to as a set. Only on the basis of simplicity are the objects of a set considered to be distinct.

A family or collection of sets are common names for a group of sets.

So,

The set has 4 numbers by substituting the numbers we get to know if the equation is true or false.

Take the equation,

4(8 - x) = 8

If x = 4,

4(8 - 4) = 8

4 × 4 = 8

16 ≠ 8

Now, If x = 5,

4(8 - 5) = 8

4 × 3 = 8

12 ≠ 8

Now, If x = 6,

4(8 - 6) = 8

4 × 2 = 8

8 = 8

And, If x = 7,

4(8 - 7) = 8

4 × 1 = 8

4 ≠ 8

Therefore, Option-C is correct that is number 6.

To know more about set visit:

https://brainly.com/question/14920396

https://brainly.com/question/30155631

If a bat catches a total of 560 bugs in 7 nights, and each night the bat catches the same number of bugs, how many bugs did the bat catch on the fifth night?

Answers

Answer:

400

Step-by-step explanation:

get the number of bugs caught in night and multiply it by five

On Thursday night Antonio watched a movie that was 1 hour and 43 minutes long. If the movie ended at the time shown on the clock below, what time did Antonio start watching the movie? Be sure to include a.m. or p.m. in your answer.

Answers

Antonio started watching the movie at 5:17pm

At what time did Antonio start the movie?

If the movie ended at 7:00pm, and we know that the movie was 1 hour and 43 minutes long, we can then subtract 1 hour and 43 minutes from 7:00pm to find out what time Antonio started the movie.

To subtract 1 hour and 43 minutes from 7:00pm:

We will first convert 7:00pm to 24-hour format which is 19:00.We can subtract 1 hour and 43 minutes which gives us:

= 19:00 - 1:43

= 17:17

As a 24-hour mode, when we convert 17:17 to 12 hours, this gives us 5:17pm.

Note: The movie ended at 7:00pm

Read more about time

brainly.com/question/26862717

#SPJ1

Use the information given in the table on the right to complete each of the following statements. Brenda is 50 inches tall. Her z-score is .

Answers

The z-score of Brenda for the mean of 49 inches and standard deviation 2 inches is equal to 0.5.

Mean is equal to 49 inches

Standard deviation is equal to 2 inches

Brenda is 50 inches tall.

To find Brenda's z-score,

Calculate the number of standard deviations that her height is away from the mean height for 7-year-olds.

z-score = (Brenda's height - Mean height) / Standard deviation

Substituting the given values, we get,

⇒ z-score = (50 - 49) / 2

⇒ z-score = 0.5

Statement 1,

Brenda's height is 0.5 standard deviations above the mean height for 7-year-olds.

Statement 2,

Approximately 68.27% of 7-year-olds are shorter than Brenda.

Using a standard normal distribution table to find the percentage of the area under the curve to the left of z = 0.5.

Therefore, Brenda's z-score for the given mean and standard deviation  is equal to 0.5.

Learn more about z-score here

brainly.com/question/12615610

#SPJ4

The above question is incomplete, the complete question is:

Use the information given in the table on the right to complete each of the following statements. Brenda is 50 inches tall. Her z-score is

Find individual value in normal distribution

Age           Mean                 Standard deviation

7years       49 inches             2 inches

a couple decide they want to have 6 children. use binomial probability to find the probability of having the same number of boys and girls.

Answers

This means that there is a [tex]31.25%[/tex] chance of having the same number of boys and girls, or roughly [tex]0.3125.[/tex]

The binomial-probability formula can be used to determine the likelihood of having a specific number of boys and girls, assuming that the odds of having either a boy or a girl are equal, which is [tex]1/2:[/tex]

[tex]C(n,k) = P(X=k) * p^k * (1-p)^{n-k}[/tex]

Where:

k is the number of successes, which is the number of boys the couple wants to have and is also equal to the number of girls because they want an equal number of each; in this case,[tex]k = 6[/tex], where n is the number of tries, which is the number of children the couple expects to have. As a result,[tex]k = n/2 = 3[/tex] and p is the success-probability, which is equal to the likelihood of having a boy or girl, which is equals to [tex]1/2[/tex]. [tex]C(n,k) = n! / (k! * (n-k)!)[/tex], where! stands for the factorial function, is the number of combinations of n things taken k at a time.

[tex]0.3125.[/tex]

When these values are added to the formula:

[tex]P(X=3) = C(6,3) × (1/2)^3 * (1/2)^(6-3) \\= 20 * (1/2)^6\\= 0.3125[/tex]

To know more about Binomial-probability visit:

https://brainly.com/question/12474772

#SPJ4

the following code segment is intended to set max equal to the maximum value among the integer variables x, y, and z. the code segment does not work as intended in all cases.

Answers

we do not have the code segment, we cannot identify which of the cases above will cause the code segment to not work as intended.

What is a conditional statement?

A conditional statement is a type of structure to express the relationship between two dependent variables. Its structure is as follows:

The code segment in question uses variables and conditional statements to determine the maximum value among three integer variables x, y, and z. However, there are cases where this code segment may not work as intended.

To determine which initial values for x, y, and z will cause this issue, we need to look at the code segment itself.

The code segment likely involves setting a variable called "max" to the value of one of the three variables (x, y, or z), and then using conditional statements to check if the other variables are larger than the current value of "max".

If they are, the value of "max" is updated to that variable.

To determine which initial values will cause issues, we need to consider cases where the conditional statements will not work as intended.

For example, if all three variables have the same value, the code segment may not correctly identify the maximum value.

Similarly, if two variables have the same value, but that value is smaller than the third variable, the code segment may not correctly identify the maximum value.

Based on the answer choices provided, it appears that the code segment may not work as intended for the initial values x = 1, y = 3, z = 2.

In this case, the initial value of "max" would be set to 1, and the conditional statements would not update the value of "max" to the correct maximum value of 3.

In conclusion, the code segment may not work as intended in cases where there are ties or when the initial values are not in order. It is important to test the code segment with different initial values to ensure that it works correctly in all cases.

The code segment in question is intended to set the variable "max" equal to the maximum value among the integer variables x, y, and z.

To demonstrate that the code segment does not work as intended in all cases, we need to evaluate each of the given initial values for x, y, and z.

1. x = 1, y = 2, z = 3: In this case, the maximum value is 3, which corresponds to the variable z.

2. x = 1, y = 3, z = 2: In this case, the maximum value is 3, which corresponds to the variable y.

3. x = 2, y = 3, z = 1: In this case, the maximum value is 3, which corresponds to the variable y.

4. x = 3, y = 2, z = 1: In this case, the maximum value is 3, which corresponds to the variable x.

Since we do not have the code segment, we cannot identify which of the cases above will cause the code segment to not work as intended.

However, you can test each case by implementing the code segment, assigning the initial values to the variables x, y, and z, and then checking if the variable "max" is set to the correct maximum value.

If any of the cases result in an incorrect "max" value, it means that the code segment does not work as intended for that particular set of initial values.

Hence, we do not have the code segment, we cannot identify which of the cases above will cause the code segment to not work as intended.

To learn more about the conditional statement visit: brainly.com/question/18152035

#SPJ4

consider the function arctan(x^2) write a partial sum for the power series which represents this function consisting of the first four non zero terms

Answers

The partial sum for the power series of arctan[tex](x^2)[/tex] consisting of the first four non-zero terms can be written as: arctan[tex](x^2) ≈ x^2 - (1/3)x^6 + (1/5)x^10 - (1/7)x^14[/tex]

The power series representation of the function f(x) = arctan[tex](x^2)[/tex] is given by:

[tex]f(x) = ∑n=0^∞ (-1)^n x^(2n+1) / (2n+1)[/tex]

To find the partial sum consisting of the first four non-zero terms, we can simply substitute n = 0, 1, 2, and 3 into the above expression, and add up the resulting terms:

[tex]f(x) ≈ x - x^3/3 + x^5/5 - x^7/7[/tex]

This is the desired partial sum for the power series representation of arctan[tex](x^2)[/tex], consisting of the first four non-zero terms. Note that as we add more terms to this sum, we get a better and better approximation to the function f(x) over a wider range of x values.

To know more about power series representation, visit the link given below:

https://brainly.com/question/29888415

#SPJ4

what is true about quantitative versus qualitative research? multiple choice all of the statements about quantitative versus qualitative research are true. none of the statements about quantitative versus qualitative research are true. quantitative research is more subjective than qualitative research quantitative research is richer than qualitative research quantitative research is more time-consuming than qualitative research

Answers

None of the statements about quantitative versus qualitative research are true.

This is because the comparison between quantitative and qualitative research is not a matter of one being objectively better or more subjective, richer, or more time-consuming than the other. They are different approaches to research, each with its strengths and limitations, and the choice between them depends on the research question, the nature of the data, and the research design.

Quantitative research is characterized by the collection of numerical data that can be analyzed using statistical methods to identify patterns, relationships, and trends. This approach is used when the research question requires an objective measurement of variables that can be quantified and compared across groups. For example, a quantitative study may examine the relationship between height and weight in a population, using statistical techniques to identify the strength of the relationship and its significance.

To know more about quantitative research,visit:

https://brainly.com/question/30472457

#SPJ11

Use the integral test to determine whether the series is convergent or divergent.

Answers

We need to find the function f(n) whose terms are the same as the series in question. We can then integrate this function from n=1 to infinity and determine if the integral is convergent or divergent. If it is convergent, then the series is convergent. If it is divergent, then the series is also divergent.

To determine whether a series is convergent or divergent using the integral test, we need to first check if the series satisfies three conditions:

1) The terms of the series are positive.

2) The terms of the series are decreasing.

3) The series has an infinite number of terms.

Assuming these conditions are satisfied, we can use the integral test which states that if the integral of the function f(x) from n=1 to infinity is convergent, then the series with terms a_n = f(n) is also convergent. Conversely, if the integral is divergent, then the series is also divergent.

Learn more about integral here:

brainly.com/question/18125359

#SPJ11

Magan invested $790 in an account paying an interest rate of 83% compounded
quarterly. Angel invested $790 in an account paying an interest rate of 8%
compounded continuously. After 7 years, how much more money would Magan have
in his account than Angel, to the nearest dollar?

Answers

Answer:3

Step-by-step explanation:

1437.9874-1444.882=3.1054

A steamer goes downstream and covers the distance between two ports in 4 hours while it covers the same distance upstream in 5 hours. If the speed of the stream is 2 km per hour, find the speed of the steamer in still water

Answers

The speed of the streamer in still water to cover the same distance in 4 hours for downstream and 5 hours in upstream in equal to 18km/hour.

Time taken by streamer in downstream to cover some distance = 4hours

Time taken by streamer in upstream to cover same distance = 5 hours

Let the speed of the streamer in still water be x km/hour.

Speed of the stream is 2 km per hour

Then ,

Speed of the streamer in downstream = ( x+ 2) km/hour

Speed of the streamer in upstream = ( x - 2) km/hour

Distance covered by streamer in down stream in 4 hours

= Distance covered by streamer in up stream in 5 hours

⇒ 4 ( x + 2) = 5( x -2)

⇒ 4x + 8 = 5x -10

⇒ 5x - 4x = 10 + 8

⇒ x = 18 km/hour.

Therefore, the speed of the streamer in still water in equal to 18km/hour.

Learn more about speed here

brainly.com/question/24147771

#SPJ4

what is the probability that maximum speed differs from the mean value by at most 2.5 standard deviations?

Answers

The answer  is that the probability that the maximum speed differs from the mean value by at most 2.5 standard deviations can be calculated using the normal distribution.

We first need to calculate the mean value and standard deviation of the speed data. Then, we can use the formula for the normal distribution to find the probability that the maximum speed falls within a certain range from the mean value.

Specifically, we can use the formula:

P(mean - 2.5SD < max speed < mean + 2.5SD) = P(z < 2.5) - P(z < -2.5)

where z is the z-score of the maximum speed, calculated as (max speed - mean)/SD.

We can find the probabilities P(z < 2.5) and P(z < -2.5) using a standard normal distribution table or calculator.

In general, if the maximum speed is normally distributed and the mean and standard deviation are known, we can use this formula to calculate the probability that the maximum speed falls within a certain range from the mean value.

To know more about probability visit:

brainly.com/question/30034780

#SPJ11

What is the probability of selecting a heart replacing then selecting a star?

Answers

The probability of selecting a heart and then a star with replacement is approximately 0.1875 or 18.75%.

Assuming that a standard deck of 52 playing cards is used, with 13 cards of each suit (including hearts) and 4 suits in total, the probability of selecting a heart on the first draw and then selecting a star (presumably meaning a card from a different suit) on the second draw with replacement is

P (heart than star)

= P (heart) × P (star)

= 13/52 × 39/52

= 507/2704

= 0.1875

where P (heart) is 13/52 is the probability of selecting a heart on the first draw (since there are 13 hearts in the deck), and P (star) is the probability of selecting a card that is not a heart on the second draw (since there are 39 non-heart cards left in the deck after the heart is replaced).

Therefore, the probability of selecting a heart and then a star with replacement is approximately 0.1875 or 18.75%.

To know more about probability here

https://brainly.com/question/30034780

#SPJ4

-- The given question is incomplete, the complete question

"What is the probability of selecting a heart and then a star with replacement?" --

z varies with y and inversely with x when z=6, x=4, and y=3

Answers

If z varies with y and inversely with and z = 6 when x = 4 and y = 3, then the value of Proportionality Constant is given by 8.

Proportion is a relation between two mathematical variables. If two variables vary directly that states if one increases another will also decrease and same for decrease.

If two variables are in inverse relation that states that if one variable increases then another decreases and if one variable decreases then another increases.  

Given that, z varies with y and inversely with x. So,

z = k*(y/x), where k is the proportionality constant.

Given that, z = 6 when x = 4 and y = 3. So,

6 = k*(3/4)

k = (6*4)/3

k = 2*4

k = 8

Hence the value of Proportionality Constant is 8.

To know more about proportionality constant here

https://brainly.com/question/27598477

#SPJ1

The question is incomplete. The complete question will be -

"z varies with y and inversely with x when z=6, x=4, and y=3. Find the value of Proportionality Constant."

An SRS of 20 orangutans is selected, and 65 cc of blood is to be drawn from each orangutan using a 100 cc syringe. In the sample, the mean volume is 64 cc and the standard deviation is 12 cc. Assume that in the population of all such procedures, the amount of blood drawn follows a Normal distribution with mean ?.
Reference: Ref 17-1
We are interested in a 95% confidence interval for the population mean volume. The margin of error associated with the confidence interval is
Answer
A. 4.64.
B. 2.68.
C. 6.84.
D. 5.62.

Answers

The margin of error associated with the 95% confidence interval for the population mean volume is: D. 5.62.

To calculate the 95% confidence interval for the population mean volume, we can use the formula:

CI = x ± (t * (s/√n))

Where CI represents the confidence interval, x is the sample mean, t is the t-score associated with the desired confidence level (95%), s is the sample standard deviation, and n is the sample size.

In this case, x = 64 cc, s = 12 cc, and n = 20 orangutans. We need to find the t-score for a 95% confidence interval with 19 degrees of freedom (n-1). Using a t-table, we find that the t-score is approximately 2.093.

Now we can calculate the margin of error:

Margin of Error = t * (s/√n) = 2.093 * (12/√20) ≈ 5.62

Therefore, the margin of error associated with the 95% confidence interval for the population mean volume is: D. 5.62.

Visit here to learn more about margin of error brainly.com/question/29419047

#SPJ11

Sketch a curve y = f(x) that satisfies: f(0) = 3, and dy/dx =-2. What is f(x)?

Answers

The curve f(x) is f(x) = -2x + 3.

Given that dy/dx = -2, we can integrate both sides with respect to x to obtain:

dy/dx = -2

dy = -2 dx

Integrating both sides, we get:

y = -2x + C

where C is the constant of integration. To find the value of C, we use the initial condition f(0) = 3:

y = -2x + C

f(0) = 3

-2(0) + C = 3

C = 3

Thus, the equation of the curve is:

y = -2x + 3

Therefore f(x) is -2x + 3

We can sketch the curve by plotting a few points. For example, when x = 0, y = 3 (as required by the initial condition). When x = 1, y = 1. When x = -1, y = 5. We can also note that the slope of the curve is always -2, meaning that the curve is a straight line with a negative slope.

Learn more about differentiation here

https://brainly.com/question/31404817

#SPJ4

you are contracted to build a children's pool at the new public pool facility. the pool must be built on a fenced off 17 by 26 feet rectangular plot. the pool must be a 77 sq ft rectangle. furthermore, the facility manager wants the border fence to be the same distance, x, from all sides of the pool, except they want to put a lifeguard chair at one end of the length, so the fence should be twice as far away on that side. what are the dimensions of the pool?

Answers

The pool dimensions are approximately 12.22 by 21.22 feet.

To start, we know that the pool must be a 77 sq ft rectangle, so we can use the formula for the area of a rectangle (length x width = area) to solve for the dimensions.

77 = length x width

Next, we know that the fenced off plot is a 17 by 26 feet rectangle. We can use this information to set up an equation for the distance between the pool and the fence:

17 - 2x = length
26 - 2x = width

We also know that the fence should be twice as far away on the end where the lifeguard chair will be located, so we can set up another equation:

17 - 4x = length (on the side with the lifeguard chair)

Now we can substitute the expressions for length and width into the formula for the area of a rectangle:

77 = (17 - 2x)(26 - 2x)

Expanding the expression on the right side, we get:

77 = 442 - 86x + 4x^2

Rearranging and simplifying, we get a quadratic equation:

4x^2 - 86x + 365 = 0

Using the quadratic formula, we can solve for x:

x = (86 +/- sqrt(86^2 - 4(4)(365))) / (2(4))

x = (86 +/- sqrt(55284)) / 8

x = (86 +/- 234.85) / 8

x = 36.86 or 2.39

We can ignore the negative value, so x = 2.39 feet.

Now we can use the equations for length and width to solve for the dimensions of the pool:

length = 17 - 2x = 17 - 2(2.39) = 12.22 feet
width = 26 - 2x (except on the side with the lifeguard chair, where it is 4x) = 26 - 2(2.39) = 21.22 feet

So the pool dimensions are approximately 12.22 by 21.22 feet.

Learn more about the dimensions here:

brainly.com/question/30503937

#SPJ11

Kiran is thinking about building a structure like this for his younger cousins to play on.
The entire structure is made out of soft foam so the children don't hurt themselves.
How much foam-would-Kiran need to build this play structure?
Kiran would need?
cubic inches of foam to build this play structure.
The entire structure is covered with vinyl so it is easy to wipe clean.
How much vinyl would Kiran need to cover this play structure?
Kiran would need?
square inches of vinyl to cover this play structure.
The foam costs 0.008¢ per in³.
The vinyl costs 0.006€ per in².
What is the total cost for all the foam and vinyl needed to build this play structure?
The cost for the foam needed is $
The cost for the vinyl needed is $
The total cost for all the foam and vinyl needed to build this play structure is $

Answers

The total cost for all the foam and vinyl needed to build the play structure would be $419.91

How to calculate the cost

Let's say the play structure is a cube with each side measuring 36 inches. The total volume of the foam needed to build this cube would be:

V = (36 in)³ = 46,656 in³

Since the structure is a cube, the surface area would be:

A = 6 × (36 in)² = 7,776 in²

The cost for the foam would be:

46,656 in³ × $0.008/in³ = $373.25

The cost for the vinyl would be:

7,776 in² × $0.006/in² = $46.66

Total cost will be:

$373.25 + $46.66 = $419.91

Learn more about cost on

https://brainly.com/question/25109150

#SPJ1

approximately what percentage of u.s. adults is overweight? select one: a. 24 percent b. 44 percent c. 64 percent d. 84 percent

Answers

According to the Centers for Disease Control and Prevention (CDC), about 73.6 percent of adults in the United States are overweight or obese. So, the correct answer is D).

This means that a majority of adults in the U.S. have a body mass index (BMI) above the healthy range, which is defined as a BMI of 18.5-24.9, which puts them at increased risk for a range of health issues, including heart disease, diabetes, and some types of cancer.

The prevalence of overweight and obesity has been increasing in the United States in recent decades, and it is a significant public health concern. Obesity is a complex issue with many causes, including genetics, environment, and lifestyle factors such as diet and physical activity levels. So, the correct answer is D).

To know more about overweight:

https://brainly.com/question/29847112

#SPJ4

--The given question is incomplete, the complete question is given

"  According to the Centers for Disease Control and Prevention (CDC), approximately what percentage of u.s. adults is overweight? select one: a. 24 percent b. 44 percent c. 64 percent d. 73.6 percent "--

Assume a company has 7 departments and they want to assign unique 3-letter codes to each department. The first two characters of the code must be capital letters (A through Z) and the last character must be a digit ( 0 through 9 ). Therefore, a department code can be any string of the form: Letter-Letter-Digit. (i) How many different department codes are possible?
(ii) How many department codes are possible if no letter appears more than once? (iii) How many department codes are possible if no letter or digit appears more than once?

Answers

(i) To find the number of different department codes possible, we need to determine the number of choices available for each character in the code.

The first two characters can be any of the 26 capital letters of the alphabet, so there are 26 choices for each of these characters. The third character must be a digit, so there are 10 choices for this character. Therefore, the total number of different department codes possible is: 26 x 26 x 10 = 6,760.



(ii) If no letter appears more than once, then we have to choose three distinct letters for the code. There are 26 choices for the first letter, 25 choices for the second letter (since we've already used one letter), and 24 choices for the third letter (since we've already used two letters).

The third character must be a digit, so there are 10 choices for this character. Therefore, the total number of different department codes possible is: 26 x 25 x 24 x 10 = 15,600.



(iii) If no letter or digit appears more than once, then we have to choose three distinct characters (either letters or digits) for the code. There are 26 choices for the first character (since it can be any letter), 25 choices for the second character (since it can be any letter except the one we chose for the first character or any digit),

and 10 choices for the third character (since it can be any digit except the one we chose for the second character). Therefore, the total number of different department codes possible is:
26 x 25 x 10 = 6,500

To know more about number click here

brainly.com/question/28210925

#SPJ11

In a research study on trends in marriage and​ family, 2​% of randomly selected parents said that they never spank their children. The​ 95% confidence interval is from 1.2​% to 2.8​% ​(n=1241​).
a) Interpret the interval in this context
A. One is 95% confident that if one were to ask every parent, between 1.2% to 2.8% of them would say they never spank their children
B. There is a 95% chance that, if one were to ask every parent, 2% of them would say they never spank their children
C. There is a 95% chance that, if one were to ask overy parent, between 1.2% to 2.8% of them would say they never spank their children
D. One is 95% confident that of the 1236 parents polled between 12% 10 2.8% of them would say they never spank their children

Answers

The given information presents the results of a research study on trends in marriage and family, which indicates that 2% of randomly selected parents said they never spank their children.

The 95% confidence interval for this proportion is from 1.2% to 2.8%, based on a sample size of 1241.

A. One is 95% confident that if one were to ask every parent, between 1.2% to 2.8% of them would say they never spank their children.

The 95% confidence interval means that if we were to take many random samples of parents and construct a 95% confidence interval for each sample, then about 95% of those intervals would contain the true proportion of parents who never spank their children.

In other words, we can be 95% confident that the true proportion of parents who never spank their children is somewhere between 1.2% and 2.8%.

Option (C) is the correct interpretation of the interval in this context, which states that there is a 95% chance that, if one were to ask every parent, between 1.2% to 2.8% of them would say they never spank their children.

This implies that the true proportion of parents who never spank their children is likely to fall between 1.2% and 2.8%.

Option (A) is also a correct interpretation of the interval, as it implies that we are 95% confident that the proportion of parents who never spank their children falls between 1.2% and 2.8%.

Option (B) is incorrect because the 95% confidence interval refers to the range of possible values for the true proportion of parents who never spank their children, not the probability that a randomly selected parent would say they never spank their children.

Option (D) is incorrect because the sample size is 1241, not 1236.

In summary, the 95% confidence interval of 1.2% to 2.8% for the proportion of parents who never spank their children indicates that we are 95% confident that the true proportion of parents who never spank their children falls between these values, if we were to ask every parent.

For similar question on probability.

https://brainly.com/question/30535529

#SPJ11

5 to the power of 2x=20

Answers

The approximate value of x in the equation 5 to the power of 2x=20 is 0.93

Calculating the value of x

From the question, we have the following parameters that can be used in our computation:

5 to the power of 2x=20

As an expression, we have

5^(2x) = 20

Take the natural logarithm of both sides

so, we have the following representation

2x = ln(20)/ln(5)

Evaluate the quotients

This gives

2x = 1.86

So, we have

x = 0.93

Hence, the value of x is 0.93

Read more about expression at

https://brainly.com/question/15775046

#SPJ1

A 8-kg block is set moving with an initial speed of 6 m/s on a rough horizontal surface. If the force of friction is 12 n, approximately how far does the block travel before it stops?.

Answers

The block will cover the distance 12m before it stops.

Newton's second law of motion:

According to Newton's second law of motion, the acceleration a

of a body of mass m is determined by the net force [tex]F_n_e_t[/tex] acting on it:

[tex]F_n_e_t[/tex]  = ma.

The mass of a block, m = 8 kg

The initial speed of the block on rough horizontal surface , u = 6 m/s

The force of friction, F = 12N

We know that, the Newton's 2nd law: [tex]F_n_e_t[/tex]  = ma.

Therefore, its final velocity is zero, v = 0

Since the only force that is acting on the body along the horizontal direction is the kinetic frictional force

[tex]F_n_e_t[/tex]  = ma. ([tex]F_n_e_t[/tex]  is acting in opposite direction to that of object motion)

a = [tex]\frac{f_n_e_t}{m}[/tex] = [tex]\frac{-12}{8} = -1.5m/s^2[/tex]

Negative sign shows that it is in the opposite direction to that of initial velocity.

By using the third kinematic equation, we get:

[tex]v^2=u^2+2as\\\\= > s = \frac{0-6^2}{2(-1.5)}\\ \\s = \frac{36}{3} = 12m[/tex]

Therefore, the block will cover the distance 12m before it stops.

Learn more about Newton's law at:

https://brainly.com/question/15280051

#SPJ4

consider the postfix expression: a-b c*(d*e-f)/(g h*k). the equivalent postfix (reverse polish notation) expression is: group of answer choices

Answers

The equivalent postfix (reverse Polish notation) expression for the infix expression "A-B+C*(DE-F)/(G+HK)" is option B: AB-CDEF-+GHK*+/.

The equivalent postfix (reverse Polish notation) expression for the given infix expression "A-B+C*(DE-F)/(G+HK)" is

AB-CDEF-+GHK*+/

Therefore, the correct answer is B), expression can be obtained by following the order of operations for postfix notation start from the left and push operands onto a stack until an operator is encountered. When an operator is encountered, pop the top two operands, perform the operation, and push the result back onto the stack.

Repeat until the end of the expression is reached, and the final result is the top value on the stack.

To know more about postfix:

https://brainly.com/question/27615498

#SPJ4

--The given question is incomplete, the complete question is given

"  Consider the postfix expression: A-B+C*(D*E-F)/(G+H*K). The equivalent postfix (reverse Polish notation) expression is:

A.

AB-C+DE*F-GH+K**/

B.

AB-CDE*F-*+GHK*+/

C.

ABC+-E*F-*+GHK*+/

D.

None of these"--

what is the price of 700g at £2 a kg

Answers

Answer:

£1.4

Step-by-step explanation:

700g = 0.7kg

0.7 x 2 = 1.4

Help pls and thank you

Answers

If right triangle BAC, m∠A = 90, m∠B = 45, and AC = 8, the length of BC is 4√2 units. So, correct option is A.

In a right triangle, the side opposite to the 90-degree angle is called the hypotenuse, and the other two sides are called the legs. In triangle BAC, AC is the hypotenuse and AB and BC are the legs. We are given that AC = 8 and ∠B = 45 degrees. We can use trigonometric ratios to find the length of BC.

The trigonometric ratio for the sine of an angle is the ratio of the length of the opposite side to the length of the hypotenuse. The trigonometric ratio for the cosine of an angle is the ratio of the length of the adjacent side to the length of the hypotenuse. Since we know that ∠B = 45 degrees, we can use the fact that sin(45) = cos(45) = 1/√2.

Let x be the length of BC. Then, we have:

sin(45) = x/8

x/8 = 1/√2

x = 8/√2

We can simplify this expression by rationalizing the denominator:

x = 8/√2 * √2/√2

x = 8√2/2

x = 4√2

So, correct option is A.

To learn more about triangle click on,

https://brainly.com/question/31437939

#SPJ1

the radius of a right circular cone is increasing at a rate of 1.8 in/s while its height is decreasing at a rate of 2.6 in/s. at what rate is the volume of the cone changing when the radius is 150 in. and the height is 128 in.? incorrect: your answer is incorrect. in3/s

Answers

The volume of the cone is changing at a rate of approximately -3368.49 cubic inches per second. The negative sign indicates that the volume is decreasing.

To find the rate at which the volume of the cone is changing, we need to use related rates and the formula for the volume of a cone, which is V = (1/3)πr²h, where V is the volume, r is the radius, and h is the height.

Given that the radius is increasing at a rate of 1.8 in/s (dr/dt = 1.8) and the height is decreasing at a rate of 2.6 in/s (dh/dt = -2.6), we need to find dV/dt when r = 150 in and h = 128 in.

First, differentiate the volume formula with respect to time (t):
dV/dt = d(1/3πr²h)/dt

Apply the product rule and chain rule:
dV/dt = (1/3)π[2rh(dr/dt) + r²(dh/dt)]

Now, substitute the given values:
dV/dt = (1/3)π[2(150)(128)(1.8) + (150)²(-2.6)]

Perform the calculations:
dV/dt ≈ (1/3)π[55296 - 58500]

dV/dt ≈ (1/3)π[-3204]

dV/dt ≈ -3368.49 in³/s

Learn more about chain rule here:

brainly.com/question/28972262

#SPJ11

an urn contains 13 white balls and 7 green balls. a sample of seven is selected at random. what is the probability that the sample contains at least one green ball?huffled standard 52-card deck of cards. what is the probability that the hand contains 4 jacks?

Answers

the probability of the sample containing at least one green ball when selecting 7 balls randomly is approximately 0.9779 or 97.79%.

To find the probability that the sample contains at least one green ball, it's easier to calculate the probability of the complementary event (not getting any green balls) and then subtract that from 1. The complementary event would be selecting all 7 balls as white balls.

1. Total balls in the urn = 13 white balls + 7 green balls = 20 balls
2. Calculate the probability of selecting 7 white balls:
  - Number of ways to choose 7 white balls out of 13 = C(13,7) = 1716
  - Number of ways to choose 7 balls out of 20 = C(20,7) = 77520
  - Probability of selecting 7 white balls = 1716 / 77520 = 0.0221267
3. Calculate the probability of getting at least one green ball:
  - Probability of at least one green ball = 1 - Probability of 7 white balls
  - Probability of at least one green ball = 1 - 0.0221267 = 0.9778733

To know more about probability Visit:

https://brainly.com/question/30034780

#SPJ11

Other Questions
Determine which of the items below would appear in the Deposits column of a bank statement and would cause an increase in the account's balance. (Check all that apply.)A returned uncollectible customer checkA note collected by the bank on behalf of the account ownerDeposits made during the monthInterest paid by the bank on the bank account balanceChecks written by the account owner Faster please!! thanks!! 30 points + brainliest!!Which is an example of simple random sampling created from each given population? find the polar coordinates of a point with Cartesian coordinates (x,y)=(53/2,5/2).Select the correct answer below:(5,5/6)(10,4/3)(5,11/6)(10,5/6)(5,4/3)(10,11/6) what christian denomination commanded the largest african-american following in the south after the civil war? quyizket edgardo is trying to lose weight. one thing he can do to help lose weight is to: group of answer choices trevor is visiting a friend out of town for the first time but feels as if he has been there before. he is experiencing: which of these terms refers to a group of two or more persons related by birth, marriage, or adoption and residing together in a household? question 10 options: maternal subfamily family instruction sw st1, 8($54) does not do anything in which of the pipeline stage? o a. if o b. id o c. ex o d. mem o e. wb o f. none of above You toss a (wrapped) sandwich to a friend leaning out of a window 10 m above you, throwing just hard enough for it to reach her. At the same instant, she drops a silver dollar to you. At what position do the dollar and the sandwich pass each other? Between 1940 and 1970 most scientists were already concerned about global warming. Which federal agency helps foreign governments with international conservation efforts?. shepherd walsh starts the month with a balance on his credit card of $800. on the 10th day of the month, he purchases $200 in clothes with his credit card. on the 15th day of the month he makes a payment on his credit card of $300. the bank charges 1.5 percent interest per month using the previous balance method. what would shepherd's finance charges be for the month? in a random sample of 65 patients undergoing a standard surgical procedure, 12 required medication for postoperative pain. in a random sample of 90 patients undergoing a new procedure, only 14 required medication. construct a 98% confidence interval for the difference in the proportions of patients needing pain medication between the old and new procedures. group of answer choices (-0.003, 0.061) Which structure listed is most like the CEO of a business, directly managers, who then instruct employees?A) hypothalamusB) anterior pituitaryC) posterior pituitaryD) brain robix has a robust incentive program that rewards employees generously for completing tasks ahead of schedule and/or under budget. they have found, however, that this plan does not motivate certain employees. of the following groups, the employees not motivated by this plan are most likely Graduate students applying for entrance to many universities must take a Miller Analogies Test. It is known that the test scores have a mean of 75 and a variance of 16. In 1990, 100 students applied for entrance into graduate school in physics. (a)[3] Find the mean and standard deviation of the sampling distribution of X. 3 A. Sivathayalan, M. Nasari, Z. Montazeri (b)[2] Find the probability that the average score of this group of students is higher than 76. (c)[3] Find the probability that the sample mean deviates from the population mean by less than 2. (d)[4] Construct a 98% confidence interval for , the true mean test score What is the purpose of the underlined part of the sentence?The girl sat in the front row and smiled for the camera. In "Where, have you gone, Charming Billy?" from Pauls memories and fantasies, you can infer that his family was Flagstaff Enterprises expected to have free cash flow in the coming year of $8 million, and this free cash flow is expected to grow at a rate of 3% per year thereafter. Flagstaff has an equity cost of capital of 13%, a debt cost of capital of 7%, and it is in the 35% corporate tax bracket. Flagstaff maintains a 0.5 debt to equity ratioA) What is the enterprise value of Flagstaff if it was an all equity firm?B) What is the enterprise value of Flagstaff as a levered firm?C) What is the present value of the interest tax shield if the firm maintain the same debt-to-equity ratio? a cylinder with a moving piston expands from an initial volume of 0.350 l against an external pressure of 1.20 atm . the expansion does 290 j of work on the surroundings. what is the final volume of the cylinder?